PT 89.S2.Q24 - The increase in minimum wage in Country X

Lime Green DotLime Green Dot Member
edited April 2021 in Logical Reasoning 1384 karma

Hey y'all--

I posted this under the Q-specific help vid, but the discussion forum here seems to get more attention sometimes, so double-posting. Promise to add helpful responses I get to my original discussion for our future LSAT progeny :3

Reference: https://7sage.com/lsat_explanations/lsat-89-section-2-question-24/

[P1] If MW↑ in Country X, the rate of UE will rapidly fall.
[P2] Should this happen, many P will have a lot more DI.
[P3] A lot of this excess DI will be spent on CG.


[CON] This rise in demand for CG will cause an ↑ in FJ to make these CG.

Initially, I didn't get why (C) was the non-weakening A/C, but it finally clicked that any ↑ in MW going on in X might actually be greater than the above-MW income some FW are getting. In another words, (C) is assuming that these (few? many?) FW who are getting "much" more than MW wouldn't also be benefitting from the uptick in MW in X.

Really just looking for some help in understanding whether (B) was also presuming something...

First, are we supposed to accept that the conclusion is referring to factory jobs only in Country X? The wording makes it sound more general--FJ across the board. That's how I took it. I get that preceding portions of the STIM are addressing what was happening specifically in X , but I don’t see where the conclusion limits itself to talking about the FJ situation in just X. Many arguments (in other questions) are wrong b/c of these types of sudden leaps (i.e., premises and conclusions talking about different, not necessarily overlapping sets), and I thought (B) was so blatantly ignoring that possibility. That’s why (B) sounded off to me at best, presumptive (without warrant) at worst–who’s to say that FJ won’t still increase overall, in line with the conclusion, regardless of where geographically these ↑ will be?

Would appreciate thoughts on this!

Comments

  • lsat5everlsat5ever Core Member
    79 karma

    I'm not positive, but I actually think the last sentence "surely his increase in demand will lead to an increase in # of jobs to meet production" is actually a sub-conclusion for the overall conclusion that an increase in minimum wage will decrease Country X's rate of unemployment. If you read it that way, you'll see that the fact that most of the consumer goods sold in Country X weakens the support between this sub conclusion / major premise and the overall conclusion because even if it is true that the # of factory jobs necessary increases, if they're out of the country it doesn't affect Country X's unemployment.

  • Lime Green DotLime Green Dot Member
    1384 karma

    Oof, you're absolutely right @lsat5ever! The last bit is sub-conclusion to the first sentence. As you mentioned, it's clear now how (B) weakens. Reminder to self: Make sure to label those conclusions/premises/sub-conclusions correctly!

    Thank you for your insight!

  • lsat5everlsat5ever Core Member
    79 karma

    of course! best of luck :)

  • Lime Green DotLime Green Dot Member
    1384 karma

    To @lsat5ever / anyone else!

    Quick follow-up: Did anyone interpret the STIM as saying the "increase in MW" was already a given? Not if it would happen, but that it was already in place? When I listened to JY's explanation, it made it sound as if the increase hadn't happened yet, but was being contemplated on the policy books. But to me it sounded as if it were already a fact and the economist is just making a causal prediction from it. Thus the increased MW = current MW?

    Assuming this is the case, does (C) get a pass for saying "much more than the current MW"? As in, even with the increase in MW in place, most FW at a few of the factories are being paid above that increased (+'current') MW?

    I know the "many factories" is problematic. Just want to be sure I see where the other holes (if any!) in this answer are.

  • yunonsieyunonsie Member
    611 karma

    I just BR'ed this PT lol. I took the stim to mean that the increase didn't happen yet. Even if (C) were true, the stimulus says "raising the minimum wage will lead to more disposable income for many people." Both of those things can happen simultaneously, so I thought the conclusion wasn't touched.

  • Lime Green DotLime Green Dot Member
    edited April 2021 1384 karma

    ooh great, thanks, @yunonsie! thinking about this question is keeping me up at night, LITERALLY lol.

    I know this may be going down a rabbit hole, but another reason I think the increased MW is already "the current MW" is b/c of the article "the"--it's definite, referring to a thing in particular, not just a general statement about MW. Had it said "an increase in the MW will lead...," I'd be inclined to believe the increase may not have happened yet.

    To your point, I think you're saying you believe that "raising the minimum wage will lead to more disposable income for many people" (STIM) and "most workers paid much more than the current MW"(C) can both be true, is that right? I think I don't disagree with that. I think they can both be true as well, b/c the "raising the MW" bit is a generalization that can apply in any situation: Like "increasing one's LSAT score leads to more open doors for many," it's just a bland, across-the-board statement. The temporal part I'm questioning is in the conclusion itself.

    (C) doesn't weaken for sure b/c of the vagueness in "many" and the fact that we don't know, relative to other jobs in X, whether the # of factories/FW involved in (C) is large enough to justify a statement pertaining to "a large segment of the working population." My problem is with the latter portion of the A/C in relation to the time aspect.

    Let's say, as you believe, the MW increase hasn't happened yet. So does (C) also fail to weaken b/c we don't know whether the "much more than current MW" will be on par with/above/below what the MW increase will eventually be?

    Let's say now that the MW increase has already taken place and is in effect such that it is the new, current MW. In this case, does (C) get spared a puncture point, b/c whatever the case may be for the rest of the working population in X, at least for some FW, the increase in place is presently above the new, current MW? This is a super trippy, convoluted explanation, but if I try to fix the clearly problematic parts of (C) while leaving the last bit in tact, I think my point will be clearer: "For all (or nearly all) workers in Country X, most are paid more than the current MW." Would this amended version weaken the argument?

  • yunonsieyunonsie Member
    611 karma

    Yeah, I would agree that version becomes a weakener!

    LSAT is tricky as hell... we've definitely seen your edited version as a correct answer choice for a weakening question before.

  • Lime Green DotLime Green Dot Member
    edited April 2021 1384 karma

    Great, thanks again @yunonsie! Yeah, I think that's why some of these Qs have good replay value... you could see them so many times and find new ways to pick 'em apart.

    Godspeed on your studies!

Sign In or Register to comment.